Difference between revisions of "2014 AIME II Problems/Problem 10"

(Solution 3)
m (Solution 3 (Roots of Unity): Added descriptor to solution title)
 
(8 intermediate revisions by 6 users not shown)
Line 4: Line 4:
  
 
==Solution 1 (long but non-bashy)==  
 
==Solution 1 (long but non-bashy)==  
 
*Commenter's note: this solution made me lose brain cells, as it is so wordy for something so simple.
 
  
 
Note that the given equality reduces to
 
Note that the given equality reduces to
Line 35: Line 33:
 
and we are done. <math>\blacksquare</math>
 
and we are done. <math>\blacksquare</math>
  
==Solution 2 (short but a little bashy)==
+
==Solution 2 (short)==
 
Assume <math>z = 2014</math>. Then
 
Assume <math>z = 2014</math>. Then
 
<cmath>\frac{1}{2014 + w} = \frac{1}{2014} + \frac{1}{w}</cmath>
 
<cmath>\frac{1}{2014 + w} = \frac{1}{2014} + \frac{1}{w}</cmath>
Line 49: Line 47:
 
Thus <math>P</math> is an isosceles triangle with area <math>\frac{1}{2}(2014 - (-1007))(2\cdot 1007\sqrt{3}) = 3021\cdot 1007\sqrt{3}</math> and <math>n \equiv 7\cdot 21\equiv \boxed{147} \pmod{1000}.</math>
 
Thus <math>P</math> is an isosceles triangle with area <math>\frac{1}{2}(2014 - (-1007))(2\cdot 1007\sqrt{3}) = 3021\cdot 1007\sqrt{3}</math> and <math>n \equiv 7\cdot 21\equiv \boxed{147} \pmod{1000}.</math>
  
==Solution 3==
+
==Solution 3 (Roots of Unity) ==
Our equation can be simplified like the following.
+
Notice that <cmath>\frac1{w+z} = \frac{w+z}{wz} \implies 0 = w^2 + wz + z^2 = \frac{w^3-z^3}{w-z}.</cmath>
<cmath>\frac{1}{w+z} = \frac{w+z}{wz}</cmath>
+
Hence, <math>w=ze^{2\pi i/3},ze^{4\pi i/3}</math>, and <math>P</math> is an equilateral triangle with circumradius <math>2014</math>. Then, <cmath>[P]=\frac{3}{2}\cdot 2014^2\cdot\sin\frac{\pi}3=3\cdot 1007^2\sqrt3,</cmath>
<cmath>wz = {(w+z)}^2</cmath>
+
and the answer is <math>3\cdot 1007^2\equiv 3\cdot 7^2\equiv\boxed{147}\pmod{1000}</math>.
<cmath>w^2 + wz + z^2 = 0</cmath>
+
 
We recognize this as the Law of Cosines with angle <math>120</math> degrees.
+
==Solution 4 (Slick)==
Our polygon is an equilateral triangle, say <math>ABC</math>, with center <math>O</math> at the origin and <math>AO=BO=CO=2014</math>. The area of <math>ABC</math> is <math>3*[ABO]=3*(1007*1007\sqrt{3})=3*1007^2*\sqrt{3}=3042147\sqrt{3}</math>. Thus, the answer is <math>\boxed{147}</math>.
+
I find that generally, the trick to these kinds of AIME problems is to interpret the problem geometrically, and that is just what I did here.  Looking at the initial equation, this seems like a difficult task, but rearranging yields a nicer equation:
 +
<cmath>\frac1{z+w}=\frac1z+\frac1w</cmath>
 +
<cmath>\frac w{z+w}=\frac wz+1</cmath>
 +
<cmath>\frac w{z+w}=\frac{w+z}z</cmath>
 +
<cmath>\frac{w-0}{w-(-z)}=\frac{(-z)-w}{(-z)-0}</cmath>
 +
We can interpret the difference of two complex numbers as a vector from one to the other, and we can interpret the quotient as a vector with an angle equal to the angle between the two vectors.  Therefore, after labeling the complex numbers with <math>W</math> (<math>w</math>), <math>V</math> (<math>-z</math>), and <math>O</math> (<math>0</math>), we can interpret the above equation to mean that the <math>\angle OWV=\angle OVW</math>, and hence triangle <math>OWV</math> is isosceles, so length <math>OW</math> = <math>OV</math>.  Rearranging the equation
 +
<cmath>\frac{w-0}{w-(-z)}=\frac{(-z)-w}{(-z)-0},</cmath>
 +
we find that
 +
<cmath>(w-0)((-z)-0)=-(w-(-z))^2.</cmath>
 +
Taking the magnitude of both sides and using the fact that <math>OW=OV\implies |w-0|=|(-z)-0|</math>, we find that
 +
<cmath>|w-0|^2=|w-(-z)|^2,</cmath>
 +
so length <math>OW=VW</math> and triangle <math>OWV</math> is equilateral.  There are only two possible <math>W</math>'s for which <math>OWV</math> is equilateral, lying on either side of <math>OV</math>. After drawing these points on the circle of radius 2014 centered at the origin, it is easy to see that <math>z</math> and the two <math>w</math>'s form an equilateral triangle (this can be verified by simple angle chasing).  Drawing a perpendicular bisector of one of the sides and using 30-60-90 triangles shows that the side length of this triangle is <math>2014\sqrt3</math> and hence its area is <math>\frac{\sqrt3(2014\sqrt3)^2}4=\boxed{147}\sqrt3+1000k\sqrt3,</math> for some integer <math>k</math>.
 +
 
 +
~SymbolicPermutation
  
-Solution by TheUltimate123
+
==Notes==
 +
This problem is killed by polar coordinates (complex numbers) . Solve using cosine-i-sine.
  
 
== See also ==
 
== See also ==
 
{{AIME box|year=2014|n=II|num-b=9|num-a=11}}
 
{{AIME box|year=2014|n=II|num-b=9|num-a=11}}
 
{{MAA Notice}}
 
{{MAA Notice}}

Latest revision as of 13:08, 21 January 2024

Problem

Let $z$ be a complex number with $|z|=2014$. Let $P$ be the polygon in the complex plane whose vertices are $z$ and every $w$ such that $\frac{1}{z+w}=\frac{1}{z}+\frac{1}{w}$. Then the area enclosed by $P$ can be written in the form $n\sqrt{3}$, where $n$ is an integer. Find the remainder when $n$ is divided by $1000$.

Solution 1 (long but non-bashy)

Note that the given equality reduces to

\[\frac{1}{w+z} = \frac{w+z}{wz}\] \[wz = {(w+z)}^2\] \[w^2 + wz + z^2 = 0\] \[\frac{w^3 - z^3}{w-z} = 0\] \[w^3 = z^3, w \neq z\]

Now, let $w = r_w e^{i \theta_w}$ and likewise for $z$. Consider circle $O$ with the origin as the center and radius 2014 on the complex plane. It is clear that $z$ must be one of the points on this circle, as $|z| = 2014$.

By DeMoivre's Theorem, the complex modulus of $w$ is cubed when $w$ is cubed. Thus $w$ must lie on $O$, since its the cube of its modulus, and thus its modulus, must be equal to $z$'s modulus.

Again, by DeMoivre's Theorem, $\theta_w$ is tripled when $w$ is cubed and likewise for $z$. For $w$, $z$, and the origin to lie on the same line, $3 \theta_w$ must be some multiple of 360 degrees apart from $3 \theta_z$ , so $\theta_w$ must differ from $\theta_z$ by some multiple of 120 degrees.

Now, without loss of generality, assume that $z$ is on the real axis. (The circle can be rotated to put $z$ in any other location.) Then there are precisely two possible distinct locations for $w$; one is obtained by going 120 degrees clockwise from $z$ about the circle and the other by moving the same amount counter-clockwise. Moving along the circle with any other multiple of 120 degrees in any direction will result in these three points.

Let the two possible locations for $w$ be $W_1$ and $W_2$ and the location of $z$ be point $Z$. Note that by symmetry, $W_1W_2Z$ is equilateral, say, with side length $x$. We know that the circumradius of this equilateral triangle is $2014$, so using the formula $\frac{abc}{4R} = [ABC]$ and that the area of an equilateral triangle with side length $s$ is $\frac{s^2\sqrt{3}}{4}$, so we have

\[\frac{x^3}{4R} = \frac{x^2\sqrt{3}}{4}\] \[x = R \sqrt{3}\] \[\frac{x^2\sqrt{3}}{4} = \frac{3R^2 \sqrt{3}}{4}\]

Since we're concerned with the non-radical part of this expression and $R = 2014$,

\[\frac{3R^2}{4} \equiv 3 \cdot 1007^2 \equiv 3 \cdot 7^2 \equiv \boxed{147} \pmod{1000}\]

and we are done. $\blacksquare$

Solution 2 (short)

Assume $z = 2014$. Then \[\frac{1}{2014 + w} = \frac{1}{2014} + \frac{1}{w}\]

\[2014w = w(2014 + w) + 2014(2014 + w)\]

\[2014w = 2014w + w^2 + 2014^2 + 2014w\]

\[0 = w^2 + 2014w + 2014^2\]

\[w = \frac{-2014 \pm \sqrt{2014^2 - 4(2014^2)}}{2} = -1007 \pm 1007\sqrt{3}i\]

Thus $P$ is an isosceles triangle with area $\frac{1}{2}(2014 - (-1007))(2\cdot 1007\sqrt{3}) = 3021\cdot 1007\sqrt{3}$ and $n \equiv 7\cdot 21\equiv \boxed{147} \pmod{1000}.$

Solution 3 (Roots of Unity)

Notice that \[\frac1{w+z} = \frac{w+z}{wz} \implies 0 = w^2 + wz + z^2 = \frac{w^3-z^3}{w-z}.\] Hence, $w=ze^{2\pi i/3},ze^{4\pi i/3}$, and $P$ is an equilateral triangle with circumradius $2014$. Then, \[[P]=\frac{3}{2}\cdot 2014^2\cdot\sin\frac{\pi}3=3\cdot 1007^2\sqrt3,\] and the answer is $3\cdot 1007^2\equiv 3\cdot 7^2\equiv\boxed{147}\pmod{1000}$.

Solution 4 (Slick)

I find that generally, the trick to these kinds of AIME problems is to interpret the problem geometrically, and that is just what I did here. Looking at the initial equation, this seems like a difficult task, but rearranging yields a nicer equation: \[\frac1{z+w}=\frac1z+\frac1w\] \[\frac w{z+w}=\frac wz+1\] \[\frac w{z+w}=\frac{w+z}z\] \[\frac{w-0}{w-(-z)}=\frac{(-z)-w}{(-z)-0}\] We can interpret the difference of two complex numbers as a vector from one to the other, and we can interpret the quotient as a vector with an angle equal to the angle between the two vectors. Therefore, after labeling the complex numbers with $W$ ($w$), $V$ ($-z$), and $O$ ($0$), we can interpret the above equation to mean that the $\angle OWV=\angle OVW$, and hence triangle $OWV$ is isosceles, so length $OW$ = $OV$. Rearranging the equation \[\frac{w-0}{w-(-z)}=\frac{(-z)-w}{(-z)-0},\] we find that \[(w-0)((-z)-0)=-(w-(-z))^2.\] Taking the magnitude of both sides and using the fact that $OW=OV\implies |w-0|=|(-z)-0|$, we find that \[|w-0|^2=|w-(-z)|^2,\] so length $OW=VW$ and triangle $OWV$ is equilateral. There are only two possible $W$'s for which $OWV$ is equilateral, lying on either side of $OV$. After drawing these points on the circle of radius 2014 centered at the origin, it is easy to see that $z$ and the two $w$'s form an equilateral triangle (this can be verified by simple angle chasing). Drawing a perpendicular bisector of one of the sides and using 30-60-90 triangles shows that the side length of this triangle is $2014\sqrt3$ and hence its area is $\frac{\sqrt3(2014\sqrt3)^2}4=\boxed{147}\sqrt3+1000k\sqrt3,$ for some integer $k$.

~SymbolicPermutation

Notes

This problem is killed by polar coordinates (complex numbers) . Solve using cosine-i-sine.

See also

2014 AIME II (ProblemsAnswer KeyResources)
Preceded by
Problem 9
Followed by
Problem 11
1 2 3 4 5 6 7 8 9 10 11 12 13 14 15
All AIME Problems and Solutions

The problems on this page are copyrighted by the Mathematical Association of America's American Mathematics Competitions. AMC logo.png